Howdy, Stranger!

It looks like you're new here. If you want to get involved, click one of these buttons!

Fake LSAT question

samcatchem500samcatchem500 Free Trial Member
edited December 2015 in Logical Reasoning 5 karma
I'm practicing, and I am having trouble with this particular question.

The correct answer is (B), however, the stimulus does not state (or imply) that more or less molecular motion is produced in "hot" climates as opposed to "cold" climates; the stimulus only states that "...molecular motion is directly proportional to the temperature..."

I could use a bit of help with attacking answer choices that expect me to make assumptions of this nature.

Thanks

Comments

  • Angelica PicklesAngelica Pickles Alum Member
    edited September 2013 28 karma
    Hey sam,
    You mean S3, right?

    We are told:
    hotter the retina, the more molecular motion, the more error prone

    From which we can indeed infer B) Animals with warmer retina are more error prone than animals with cooler retina.
  • samcatchem500samcatchem500 Free Trial Member
    5 karma
    Angelica!
    A friend of mine's sent me a group of PDFs he used when he took the LSATs. The fact that your first question asked if I was referring to section 3 lets me know that these files might be seriously flawed. The stimulus on my version did not mention anything regarding "hot" or "cold". Thank You So Much Angelica for responding, and letting me know that these questions are not correlating with the official PrepTests.
Sign In or Register to comment.